To win democratic elections that are not fully subsidized by the government, nonwealthy candidates must be supported ...

LSATQz on November 14, 2019

Why is C incorrect?

Could someone please explain why C is incorrect?

Reply
Create a free account to read and take part in forum discussions.

Already have an account? log in

Skylar on November 14, 2019

@LSATQz Happy to help!

(C) is incorrect because it is irrelevant to the argument's focus. The passage is only looking at "democratic elections that are not fully subsidized by the government," so information about about government-subsidized elections in democracies like those discussed in (C) have no effect on those discussed in the passage and could be consistent with the argument.

Does that help? Let us know if you have any additional questions!